For which $m$ does $abequiv -1pmod{m}$ imply $a+bequiv 0pmod{m}$ for positive integers $a$ and $b$?












1












$begingroup$


This is pretty easy to show for $m=24,$ and I'm pretty sure if $abequiv -1pmod{p^{n}}$ does not imply $a+bequiv 0pmod{p^{n}}$ then $abequiv -1pmod{p^{n+k}}$ doesn't imply $a+bequiv 0pmod{p^{n+k}}$ for any positive integer $k$ either. I haven't managed to find any larger numbers than $24,$ so how would I go about proving there exist no $m>24$? If there do exist $m>24,$ how do I show it (and how do I generate $m$ which satisfy this condition)?










share|cite|improve this question











$endgroup$












  • $begingroup$
    What about the title? It says $a+b equiv 0 mod m$; but the text addresses $a + b equiv -1 mod m$. Both engaging cases. Do you mean one or both?
    $endgroup$
    – Robert Lewis
    Jan 10 at 3:04






  • 1




    $begingroup$
    So sorry, that was a typo! I meant $a+bequiv 0pmod{m}.$
    $endgroup$
    – P-addict
    Jan 10 at 3:06










  • $begingroup$
    Thanks for the speedy correction!
    $endgroup$
    – Robert Lewis
    Jan 10 at 3:06










  • $begingroup$
    That's equivalent to: $,a,$ invertible $Rightarrow,a^{-1}equiv a [!iff a^2equiv 1] $ Is that what you intend?
    $endgroup$
    – Bill Dubuque
    Jan 10 at 3:12












  • $begingroup$
    Yes, I think that's what I was going for.
    $endgroup$
    – P-addict
    Jan 10 at 3:28
















1












$begingroup$


This is pretty easy to show for $m=24,$ and I'm pretty sure if $abequiv -1pmod{p^{n}}$ does not imply $a+bequiv 0pmod{p^{n}}$ then $abequiv -1pmod{p^{n+k}}$ doesn't imply $a+bequiv 0pmod{p^{n+k}}$ for any positive integer $k$ either. I haven't managed to find any larger numbers than $24,$ so how would I go about proving there exist no $m>24$? If there do exist $m>24,$ how do I show it (and how do I generate $m$ which satisfy this condition)?










share|cite|improve this question











$endgroup$












  • $begingroup$
    What about the title? It says $a+b equiv 0 mod m$; but the text addresses $a + b equiv -1 mod m$. Both engaging cases. Do you mean one or both?
    $endgroup$
    – Robert Lewis
    Jan 10 at 3:04






  • 1




    $begingroup$
    So sorry, that was a typo! I meant $a+bequiv 0pmod{m}.$
    $endgroup$
    – P-addict
    Jan 10 at 3:06










  • $begingroup$
    Thanks for the speedy correction!
    $endgroup$
    – Robert Lewis
    Jan 10 at 3:06










  • $begingroup$
    That's equivalent to: $,a,$ invertible $Rightarrow,a^{-1}equiv a [!iff a^2equiv 1] $ Is that what you intend?
    $endgroup$
    – Bill Dubuque
    Jan 10 at 3:12












  • $begingroup$
    Yes, I think that's what I was going for.
    $endgroup$
    – P-addict
    Jan 10 at 3:28














1












1








1





$begingroup$


This is pretty easy to show for $m=24,$ and I'm pretty sure if $abequiv -1pmod{p^{n}}$ does not imply $a+bequiv 0pmod{p^{n}}$ then $abequiv -1pmod{p^{n+k}}$ doesn't imply $a+bequiv 0pmod{p^{n+k}}$ for any positive integer $k$ either. I haven't managed to find any larger numbers than $24,$ so how would I go about proving there exist no $m>24$? If there do exist $m>24,$ how do I show it (and how do I generate $m$ which satisfy this condition)?










share|cite|improve this question











$endgroup$




This is pretty easy to show for $m=24,$ and I'm pretty sure if $abequiv -1pmod{p^{n}}$ does not imply $a+bequiv 0pmod{p^{n}}$ then $abequiv -1pmod{p^{n+k}}$ doesn't imply $a+bequiv 0pmod{p^{n+k}}$ for any positive integer $k$ either. I haven't managed to find any larger numbers than $24,$ so how would I go about proving there exist no $m>24$? If there do exist $m>24,$ how do I show it (and how do I generate $m$ which satisfy this condition)?







number-theory modular-arithmetic






share|cite|improve this question















share|cite|improve this question













share|cite|improve this question




share|cite|improve this question








edited Jan 10 at 3:05







P-addict

















asked Jan 10 at 2:59









P-addictP-addict

284




284












  • $begingroup$
    What about the title? It says $a+b equiv 0 mod m$; but the text addresses $a + b equiv -1 mod m$. Both engaging cases. Do you mean one or both?
    $endgroup$
    – Robert Lewis
    Jan 10 at 3:04






  • 1




    $begingroup$
    So sorry, that was a typo! I meant $a+bequiv 0pmod{m}.$
    $endgroup$
    – P-addict
    Jan 10 at 3:06










  • $begingroup$
    Thanks for the speedy correction!
    $endgroup$
    – Robert Lewis
    Jan 10 at 3:06










  • $begingroup$
    That's equivalent to: $,a,$ invertible $Rightarrow,a^{-1}equiv a [!iff a^2equiv 1] $ Is that what you intend?
    $endgroup$
    – Bill Dubuque
    Jan 10 at 3:12












  • $begingroup$
    Yes, I think that's what I was going for.
    $endgroup$
    – P-addict
    Jan 10 at 3:28


















  • $begingroup$
    What about the title? It says $a+b equiv 0 mod m$; but the text addresses $a + b equiv -1 mod m$. Both engaging cases. Do you mean one or both?
    $endgroup$
    – Robert Lewis
    Jan 10 at 3:04






  • 1




    $begingroup$
    So sorry, that was a typo! I meant $a+bequiv 0pmod{m}.$
    $endgroup$
    – P-addict
    Jan 10 at 3:06










  • $begingroup$
    Thanks for the speedy correction!
    $endgroup$
    – Robert Lewis
    Jan 10 at 3:06










  • $begingroup$
    That's equivalent to: $,a,$ invertible $Rightarrow,a^{-1}equiv a [!iff a^2equiv 1] $ Is that what you intend?
    $endgroup$
    – Bill Dubuque
    Jan 10 at 3:12












  • $begingroup$
    Yes, I think that's what I was going for.
    $endgroup$
    – P-addict
    Jan 10 at 3:28
















$begingroup$
What about the title? It says $a+b equiv 0 mod m$; but the text addresses $a + b equiv -1 mod m$. Both engaging cases. Do you mean one or both?
$endgroup$
– Robert Lewis
Jan 10 at 3:04




$begingroup$
What about the title? It says $a+b equiv 0 mod m$; but the text addresses $a + b equiv -1 mod m$. Both engaging cases. Do you mean one or both?
$endgroup$
– Robert Lewis
Jan 10 at 3:04




1




1




$begingroup$
So sorry, that was a typo! I meant $a+bequiv 0pmod{m}.$
$endgroup$
– P-addict
Jan 10 at 3:06




$begingroup$
So sorry, that was a typo! I meant $a+bequiv 0pmod{m}.$
$endgroup$
– P-addict
Jan 10 at 3:06












$begingroup$
Thanks for the speedy correction!
$endgroup$
– Robert Lewis
Jan 10 at 3:06




$begingroup$
Thanks for the speedy correction!
$endgroup$
– Robert Lewis
Jan 10 at 3:06












$begingroup$
That's equivalent to: $,a,$ invertible $Rightarrow,a^{-1}equiv a [!iff a^2equiv 1] $ Is that what you intend?
$endgroup$
– Bill Dubuque
Jan 10 at 3:12






$begingroup$
That's equivalent to: $,a,$ invertible $Rightarrow,a^{-1}equiv a [!iff a^2equiv 1] $ Is that what you intend?
$endgroup$
– Bill Dubuque
Jan 10 at 3:12














$begingroup$
Yes, I think that's what I was going for.
$endgroup$
– P-addict
Jan 10 at 3:28




$begingroup$
Yes, I think that's what I was going for.
$endgroup$
– P-addict
Jan 10 at 3:28










1 Answer
1






active

oldest

votes


















0












$begingroup$

Suppose that both conditions hold. Then $aequiv -b pmod{m}$ and thus $$a^2 equiv -ab equiv 1 pmod{m},$$



and so $a^2 - 1 = mx.$ So, it suffices for there to be a number which is not its own inverse. For this, the Chinese Remainder Theorem is of great use.



It's clear that, for any prime besides $2, 3,$ there is some number which is not its own inverse (just note that if $a^2 - 1 = px,$ then $(a-1)(a+1) = px,$ and so either $aequiv 1pmod{p}$ or $aequiv -1pmod{p}.$)



So, suppose that some prime $p > 3$ divides $m,$ and that $p^n$ is largest power of $p$ that divides $m.$ Then, find a residue mod $m$ which is congruent to $2$ mod $p^n$ but $1$ modulo every other prime power dividing $m.$ This residue will be invertible as it is relatively prime to $m$, and thus have a unique inverse mod $m.$ But it clearly cannot be its own inverse, as it is congruent to $2$ mod $p^n$ but $2$ is not its own inverse mod $p^n$ since $p > 3,$ and thus the equation $a+b equiv 0 pmod{m}$ does not hold, as if it did the above computations would show that $a$ was its own inverse.






share|cite|improve this answer









$endgroup$













  • $begingroup$
    You haven't specifically answered the question, i.e. "For which $m$...."
    $endgroup$
    – Bill Dubuque
    Jan 10 at 4:05












  • $begingroup$
    @BillDubuque I have--those m such that no prime $p > 3$ divides $m.$
    $endgroup$
    – Michael Barz
    Jan 10 at 23:34










  • $begingroup$
    That's not correct, e.g. $,3^{large 2}!notequiv 1pmod{!16}, 2^{large 2}!notequiv 1pmod{!9} $
    $endgroup$
    – Bill Dubuque
    Jan 10 at 23:40













Your Answer





StackExchange.ifUsing("editor", function () {
return StackExchange.using("mathjaxEditing", function () {
StackExchange.MarkdownEditor.creationCallbacks.add(function (editor, postfix) {
StackExchange.mathjaxEditing.prepareWmdForMathJax(editor, postfix, [["$", "$"], ["\\(","\\)"]]);
});
});
}, "mathjax-editing");

StackExchange.ready(function() {
var channelOptions = {
tags: "".split(" "),
id: "69"
};
initTagRenderer("".split(" "), "".split(" "), channelOptions);

StackExchange.using("externalEditor", function() {
// Have to fire editor after snippets, if snippets enabled
if (StackExchange.settings.snippets.snippetsEnabled) {
StackExchange.using("snippets", function() {
createEditor();
});
}
else {
createEditor();
}
});

function createEditor() {
StackExchange.prepareEditor({
heartbeatType: 'answer',
autoActivateHeartbeat: false,
convertImagesToLinks: true,
noModals: true,
showLowRepImageUploadWarning: true,
reputationToPostImages: 10,
bindNavPrevention: true,
postfix: "",
imageUploader: {
brandingHtml: "Powered by u003ca class="icon-imgur-white" href="https://imgur.com/"u003eu003c/au003e",
contentPolicyHtml: "User contributions licensed under u003ca href="https://creativecommons.org/licenses/by-sa/3.0/"u003ecc by-sa 3.0 with attribution requiredu003c/au003e u003ca href="https://stackoverflow.com/legal/content-policy"u003e(content policy)u003c/au003e",
allowUrls: true
},
noCode: true, onDemand: true,
discardSelector: ".discard-answer"
,immediatelyShowMarkdownHelp:true
});


}
});














draft saved

draft discarded


















StackExchange.ready(
function () {
StackExchange.openid.initPostLogin('.new-post-login', 'https%3a%2f%2fmath.stackexchange.com%2fquestions%2f3068176%2ffor-which-m-does-ab-equiv-1-pmodm-imply-ab-equiv-0-pmodm-for-positiv%23new-answer', 'question_page');
}
);

Post as a guest















Required, but never shown

























1 Answer
1






active

oldest

votes








1 Answer
1






active

oldest

votes









active

oldest

votes






active

oldest

votes









0












$begingroup$

Suppose that both conditions hold. Then $aequiv -b pmod{m}$ and thus $$a^2 equiv -ab equiv 1 pmod{m},$$



and so $a^2 - 1 = mx.$ So, it suffices for there to be a number which is not its own inverse. For this, the Chinese Remainder Theorem is of great use.



It's clear that, for any prime besides $2, 3,$ there is some number which is not its own inverse (just note that if $a^2 - 1 = px,$ then $(a-1)(a+1) = px,$ and so either $aequiv 1pmod{p}$ or $aequiv -1pmod{p}.$)



So, suppose that some prime $p > 3$ divides $m,$ and that $p^n$ is largest power of $p$ that divides $m.$ Then, find a residue mod $m$ which is congruent to $2$ mod $p^n$ but $1$ modulo every other prime power dividing $m.$ This residue will be invertible as it is relatively prime to $m$, and thus have a unique inverse mod $m.$ But it clearly cannot be its own inverse, as it is congruent to $2$ mod $p^n$ but $2$ is not its own inverse mod $p^n$ since $p > 3,$ and thus the equation $a+b equiv 0 pmod{m}$ does not hold, as if it did the above computations would show that $a$ was its own inverse.






share|cite|improve this answer









$endgroup$













  • $begingroup$
    You haven't specifically answered the question, i.e. "For which $m$...."
    $endgroup$
    – Bill Dubuque
    Jan 10 at 4:05












  • $begingroup$
    @BillDubuque I have--those m such that no prime $p > 3$ divides $m.$
    $endgroup$
    – Michael Barz
    Jan 10 at 23:34










  • $begingroup$
    That's not correct, e.g. $,3^{large 2}!notequiv 1pmod{!16}, 2^{large 2}!notequiv 1pmod{!9} $
    $endgroup$
    – Bill Dubuque
    Jan 10 at 23:40


















0












$begingroup$

Suppose that both conditions hold. Then $aequiv -b pmod{m}$ and thus $$a^2 equiv -ab equiv 1 pmod{m},$$



and so $a^2 - 1 = mx.$ So, it suffices for there to be a number which is not its own inverse. For this, the Chinese Remainder Theorem is of great use.



It's clear that, for any prime besides $2, 3,$ there is some number which is not its own inverse (just note that if $a^2 - 1 = px,$ then $(a-1)(a+1) = px,$ and so either $aequiv 1pmod{p}$ or $aequiv -1pmod{p}.$)



So, suppose that some prime $p > 3$ divides $m,$ and that $p^n$ is largest power of $p$ that divides $m.$ Then, find a residue mod $m$ which is congruent to $2$ mod $p^n$ but $1$ modulo every other prime power dividing $m.$ This residue will be invertible as it is relatively prime to $m$, and thus have a unique inverse mod $m.$ But it clearly cannot be its own inverse, as it is congruent to $2$ mod $p^n$ but $2$ is not its own inverse mod $p^n$ since $p > 3,$ and thus the equation $a+b equiv 0 pmod{m}$ does not hold, as if it did the above computations would show that $a$ was its own inverse.






share|cite|improve this answer









$endgroup$













  • $begingroup$
    You haven't specifically answered the question, i.e. "For which $m$...."
    $endgroup$
    – Bill Dubuque
    Jan 10 at 4:05












  • $begingroup$
    @BillDubuque I have--those m such that no prime $p > 3$ divides $m.$
    $endgroup$
    – Michael Barz
    Jan 10 at 23:34










  • $begingroup$
    That's not correct, e.g. $,3^{large 2}!notequiv 1pmod{!16}, 2^{large 2}!notequiv 1pmod{!9} $
    $endgroup$
    – Bill Dubuque
    Jan 10 at 23:40
















0












0








0





$begingroup$

Suppose that both conditions hold. Then $aequiv -b pmod{m}$ and thus $$a^2 equiv -ab equiv 1 pmod{m},$$



and so $a^2 - 1 = mx.$ So, it suffices for there to be a number which is not its own inverse. For this, the Chinese Remainder Theorem is of great use.



It's clear that, for any prime besides $2, 3,$ there is some number which is not its own inverse (just note that if $a^2 - 1 = px,$ then $(a-1)(a+1) = px,$ and so either $aequiv 1pmod{p}$ or $aequiv -1pmod{p}.$)



So, suppose that some prime $p > 3$ divides $m,$ and that $p^n$ is largest power of $p$ that divides $m.$ Then, find a residue mod $m$ which is congruent to $2$ mod $p^n$ but $1$ modulo every other prime power dividing $m.$ This residue will be invertible as it is relatively prime to $m$, and thus have a unique inverse mod $m.$ But it clearly cannot be its own inverse, as it is congruent to $2$ mod $p^n$ but $2$ is not its own inverse mod $p^n$ since $p > 3,$ and thus the equation $a+b equiv 0 pmod{m}$ does not hold, as if it did the above computations would show that $a$ was its own inverse.






share|cite|improve this answer









$endgroup$



Suppose that both conditions hold. Then $aequiv -b pmod{m}$ and thus $$a^2 equiv -ab equiv 1 pmod{m},$$



and so $a^2 - 1 = mx.$ So, it suffices for there to be a number which is not its own inverse. For this, the Chinese Remainder Theorem is of great use.



It's clear that, for any prime besides $2, 3,$ there is some number which is not its own inverse (just note that if $a^2 - 1 = px,$ then $(a-1)(a+1) = px,$ and so either $aequiv 1pmod{p}$ or $aequiv -1pmod{p}.$)



So, suppose that some prime $p > 3$ divides $m,$ and that $p^n$ is largest power of $p$ that divides $m.$ Then, find a residue mod $m$ which is congruent to $2$ mod $p^n$ but $1$ modulo every other prime power dividing $m.$ This residue will be invertible as it is relatively prime to $m$, and thus have a unique inverse mod $m.$ But it clearly cannot be its own inverse, as it is congruent to $2$ mod $p^n$ but $2$ is not its own inverse mod $p^n$ since $p > 3,$ and thus the equation $a+b equiv 0 pmod{m}$ does not hold, as if it did the above computations would show that $a$ was its own inverse.







share|cite|improve this answer












share|cite|improve this answer



share|cite|improve this answer










answered Jan 10 at 3:19









Michael BarzMichael Barz

915




915












  • $begingroup$
    You haven't specifically answered the question, i.e. "For which $m$...."
    $endgroup$
    – Bill Dubuque
    Jan 10 at 4:05












  • $begingroup$
    @BillDubuque I have--those m such that no prime $p > 3$ divides $m.$
    $endgroup$
    – Michael Barz
    Jan 10 at 23:34










  • $begingroup$
    That's not correct, e.g. $,3^{large 2}!notequiv 1pmod{!16}, 2^{large 2}!notequiv 1pmod{!9} $
    $endgroup$
    – Bill Dubuque
    Jan 10 at 23:40




















  • $begingroup$
    You haven't specifically answered the question, i.e. "For which $m$...."
    $endgroup$
    – Bill Dubuque
    Jan 10 at 4:05












  • $begingroup$
    @BillDubuque I have--those m such that no prime $p > 3$ divides $m.$
    $endgroup$
    – Michael Barz
    Jan 10 at 23:34










  • $begingroup$
    That's not correct, e.g. $,3^{large 2}!notequiv 1pmod{!16}, 2^{large 2}!notequiv 1pmod{!9} $
    $endgroup$
    – Bill Dubuque
    Jan 10 at 23:40


















$begingroup$
You haven't specifically answered the question, i.e. "For which $m$...."
$endgroup$
– Bill Dubuque
Jan 10 at 4:05






$begingroup$
You haven't specifically answered the question, i.e. "For which $m$...."
$endgroup$
– Bill Dubuque
Jan 10 at 4:05














$begingroup$
@BillDubuque I have--those m such that no prime $p > 3$ divides $m.$
$endgroup$
– Michael Barz
Jan 10 at 23:34




$begingroup$
@BillDubuque I have--those m such that no prime $p > 3$ divides $m.$
$endgroup$
– Michael Barz
Jan 10 at 23:34












$begingroup$
That's not correct, e.g. $,3^{large 2}!notequiv 1pmod{!16}, 2^{large 2}!notequiv 1pmod{!9} $
$endgroup$
– Bill Dubuque
Jan 10 at 23:40






$begingroup$
That's not correct, e.g. $,3^{large 2}!notequiv 1pmod{!16}, 2^{large 2}!notequiv 1pmod{!9} $
$endgroup$
– Bill Dubuque
Jan 10 at 23:40




















draft saved

draft discarded




















































Thanks for contributing an answer to Mathematics Stack Exchange!


  • Please be sure to answer the question. Provide details and share your research!

But avoid



  • Asking for help, clarification, or responding to other answers.

  • Making statements based on opinion; back them up with references or personal experience.


Use MathJax to format equations. MathJax reference.


To learn more, see our tips on writing great answers.




draft saved


draft discarded














StackExchange.ready(
function () {
StackExchange.openid.initPostLogin('.new-post-login', 'https%3a%2f%2fmath.stackexchange.com%2fquestions%2f3068176%2ffor-which-m-does-ab-equiv-1-pmodm-imply-ab-equiv-0-pmodm-for-positiv%23new-answer', 'question_page');
}
);

Post as a guest















Required, but never shown





















































Required, but never shown














Required, but never shown












Required, but never shown







Required, but never shown

































Required, but never shown














Required, but never shown












Required, but never shown







Required, but never shown







Popular posts from this blog

Can a sorcerer learn a 5th-level spell early by creating spell slots using the Font of Magic feature?

Does disintegrating a polymorphed enemy still kill it after the 2018 errata?

A Topological Invariant for $pi_3(U(n))$